LSAT and Law School Admissions Forum

Get expert LSAT preparation and law school admissions advice from PowerScore Test Preparation.

 Administrator
PowerScore Staff
  • PowerScore Staff
  • Posts: 8917
  • Joined: Feb 02, 2011
|
#39216
Complete Question Explanation
(The complete setup for this game can be found here: lsat/viewtopic.php?t=12911)

The correct answer choice is (E)

If one of the rugs contains only P, we need to conform to the 3-1-1 distribution (the 2-2-1 distribution requires P to be used in a multicolored rug along with O).

Given the conditional relationship between O and P established by the second rule, we can safely assume that O is not used in any of the rugs (if O were used, then P would need to be used in the same rug as O). Thus:
PT74 - Game_#3_#13_diagram 1.png
Now that we know which thread color is not used, we need to determine how the remaining four colors—F, T, W, and Y—are distributed among the two remaining rugs. From the first rule, we know that W must be in a group with two other colors, so W will end up in the multicolored rug. We also need to split F and T between two separate rugs in compliance with the third rule. While we cannot determine the particular rugs in which F and T will be used (solid or multicolored), we can Hurdle this Uncertainty by using Dual Options as shown below. The remaining thread color—Y—must be used in the multicolored rug:
PT74 - Game_#3_#13_diagram 2.png
The resulting setup validates answer choice (E) as the correct answer choice to this Must Be True question.

Answer choice (A) is not necessarily true, because F could be used in the multicolored rug.

Answer choice (B) is not necessarily true, because T could be used in the multicolored rug.

Answer choice (C) must be false, because Y must be used in the multicolored rug.

Answer choice (D) is incorrect, because F and W need not be used together in any of the rugs.

Answer choice (E) is the correct answer choice.
You do not have the required permissions to view the files attached to this post.
 cnoury1221
  • Posts: 15
  • Joined: Jun 24, 2019
|
#65858
Hello,

The explanation for this question begins with the point that because one of the rugs will contain only P, the 2-1-1 distribution will be used.. but then the 3-1-1 distribution is used to get the right answer. Why not use the 2-1-1 distribution here?

Thank you!
Carolyn
 George George
PowerScore Staff
  • PowerScore Staff
  • Posts: 48
  • Joined: Jun 07, 2019
|
#65862
@cnoury1221 Good catch! Looks like a typo in the original explanation. I'll update it right now.

Get the most out of your LSAT Prep Plus subscription.

Analyze and track your performance with our Testing and Analytics Package.